¿Cómo derivar el resultado del efecto Aharonov-Bohm?

En las derivaciones de la fase Aharonov-Bohm , se menciona directamente que debido a la introducción del vector potencial A , se introduce una fase adicional en la función de onda para el caso A 0 es decir

ψ ( A 0 ) = Exp ( yo φ ) ψ ( A = 0 ) ,

dónde

φ = q PAG A d X .

Cómo derivarlo de la siguiente ecuación de Schordinger

[ 1 2 metro ( i mi A ) 2 + V ( r ) ] ψ = ϵ ψ .

Traté de tomar los términos que contienen A a la derecha y tratar la ecuación como una ecuación no homogénea pero se vuelve tedioso. ¿Cuál es la forma simple y directa?

Respuestas (2)

Primero, estableceré mi = 1 por simplicidad.

Dejar ψ 0 denotemos la función de onda que satisface la ecuación libre de Schrödinger:

(1) i ψ 0 t = 1 2 metro 2 ψ 0 + V ψ 0
Además, deja ψ sea ​​la función de onda que obedece a la ecuación de Schrödinger para un potencial vectorial que no desaparece A :
(2) i ψ t = 1 2 metro ( i A ) 2 ψ + V ψ
Escribamos ahora:
ψ = Exp ( i γ A d yo ) ψ 0
dónde γ es un camino desde algún punto arbitrario X 0 a algún otro punto X 1 . Entonces podemos escribir:
( i A ) 2 ψ = Exp ( i γ A d yo ) 2 ψ 0
Sustituyendo esta expresión en la ecuación ( 2 ) da ecuación ( 1 ) . Esto implica que la función de onda de una partícula cargada eléctricamente que viaja a través del espacio donde A 0 ganará una fase adicional.

Sabemos que la función de onda en el punto q (ver la figura a continuación) es el resultado de la superposición cuántica, es decir, podemos escribir:

ψ q = ψ ( X , γ 1 ) + ψ ( X , γ 2 ) = Exp ( i γ 1 A d yo ) ψ 0 ( X , γ 1 ) + Exp ( i γ 2 A d yo ) ψ 0 ( X , γ 2 ) = Exp ( i γ 2 A d yo ) ( Exp ( i γ 1 A d yo i γ 2 A d yo ) ψ 0 ( X , γ 1 ) + ψ 0 ( X , γ 2 ) )
Podemos usar el teorema de Stoke en el primer término dentro de los paréntesis, porque γ 1 γ 2 es un camino cerrado:
γ 1 A d yo γ 2 A d yo = B d S = F
dónde F es el flujo magnético total debido al solenoide a través de una superficie definida por el límite cerrado γ 2 γ 1 . La función de onda en q ahora se puede escribir como:
ψ q = Exp ( i γ 2 A d yo ) ( Exp ( i F ) ψ 0 ( X , γ 1 ) + ψ 0 ( X , γ 2 ) )
Esto muestra que la diferencia de fase relativa y, por lo tanto, el patrón de interferencia, depende del flujo magnético debido al solenoide. Este es el efecto Aharonov-Bohm.

ingrese la descripción de la imagen aquí

:- la respuesta es buena. Pero ya asumiste que el ψ ( A 0 ) es de una forma particular. ¿Podemos hacerlo mejor que esto?
@ user38579 No conozco un procedimiento "mejor".

Para simplificar el problema, podemos despreciar el término de energía potencial V ( r ) , ya que es simplemente irrelevante para nuestra derivación. Entonces escribimos el hamiltoniano como

H = 1 2 ( i X A ) 2 .
El estado fundamental viene dado por la minimización de la energía. Como el hamiltoniano es un cuadrado de ( i X A ) , por lo que se minimiza cuando ( i X A ) = 0 . Lo que significa que en el estado fundamental, aproximadamente tenemos
( i X A ) ψ = 0.
Si solo nos preocupamos por la configuración de fase de la función de onda, podemos escribir ψ mi i ϕ , y sustituimos en la ecuación anterior,
( X ϕ A ) mi i ϕ = 0 ,
lo que significa X ϕ = A , y su solución es ϕ = A d X .

Hola Everett. Perdón por el golpe en esta respuesta anterior. ¿No le preocupa que \psi definido de esta manera no tenga un solo valor a menos que el flujo sea un número entero? Encontré una discusión similar de Berry en este documento iopscience.iop.org/article/10.1088/0143-0807/1/3/008
Hola Ryan, buen comentario. Tienes razón, ψ no tiene un solo valor, lo que significa que el frente de onda no está definido globalmente en presencia de un flujo no entero. Creo que una mejor manera de formular el efecto AB es usar la integral de trayectoria, que no se basa en un frente de onda definido globalmente.
¡He ido bastante lejos en esta madriguera de conejo desde ayer! Me parece un problema fascinante. El documento original de Aharonov-Bohm incluso habla de esto y, de hecho, dieron una solución de valor único. La derivación más hermosa (de Berry) que encontré de esta solución en realidad sigue su truco de transformaciones de calibre singular, pero en un resumen de la expansión angular habitual es.muni.cz/el/sci/jaro2015/F8592/um/Berry. pdf _ Parece coincidir con la imagen integral de la ruta de sumar las formas en que la ruta puede rodear el flujo, pero no sé cómo relacionarlas directamente.
@RyanThorngren Ya veo. Gracias por señalar el buen artículo de Berry. Lo que obtuve del artículo es que las dos trayectorias con menos remolinos dominan la integral de trayectoria ya que acumulan la mayor cantidad de acciones estacionarias, por lo que es justo aproximar la estructura de fase mediante la transformación de calibre singular. Pero si rotamos alrededor del flujo, esos caminos sublíderes se convertirán en líderes, para solucionar el problema del valor no único. Siento que el flujo no entero conduce a una anomalía cuántica en la rotación SO (2), de modo que se permite que el momento angular tome valores fraccionarios (¿fraccionamiento de simetría?)
¡Sí! Como sabes, hay una bomba de momento angular en el problema simplificado de una partícula en un círculo con un flujo dentro. Creo que algo similar está pasando aquí. Es algo anómalo porque quiere estar al borde de una bomba 1d Thouless. Creo que en realidad puedes ver algo así aquí, la forma en que la dislocación se "bombea" a lo largo de la dirección de dispersión hacia adelante a medida que el flujo varía de 0 a 2pi.